251,31 rounded to the nearest thousand

Answers

Answer 1

Answer:

25000

Step-by-step explanation:

hkyfjntrdhjiyp[hiut0yitrjiypogk.j,rf]kirt6otykkopmdtopmopijmt7oijmuopr6[pj[pgtjkl[pktgp[jkvgpikpg;jgkp[j[fc


Related Questions

What is “a/b -c+d” if “a=7/8, b=-7/16, c=.8, and d=1/4”

Answers

Answer:

(7-8) / (-7/16) = -2 -(.8) + (1/4) = -.55 -2+ -.55= -2.55

Step-by-step explanation:

Work out the perimeter of the shaded shape. 7m 4m 1m 4m 14m​

Answers

The parameter of the shaded shape is 50 meters

The parameter of any shape is sum of all sides

Measurements of some of the sides are given and some needs to be find out so the calculations are -

first side = 7m

second side = 14m

third side is equal to first side so= 7m,

fourth side = 1m

fifth side is equal to seventh side =4m

sixth side is second side- fourth side- eighth side = 14- 1-4= 9m

seventh side=4m

eighth side =4m

parameter of the shape = sum of all side = 7+ 14+ 7+ 1+4+9+4+4

                                     =50 m

So, the parameter of the shaded shape is 50 meters

To know more about parameter - https://brainly.com/question/13566907

#SPJ9

The domain of f(x) is all real numbers & satisfies f(x+1)=2f(x).
For [tex] \sf x \in(0,1],[/tex] f(x)=x(x+1). For value [tex]\sf x \in( - \infty ,m],[/tex] f(x)[tex] \geq \dfrac{ - 8}{9}. [/tex] What is the value of m?​

Answers

The domain of f(x) is all real numbers & satisfies function f(x+1)=2f(x).

For  x∈(0,1], the function f(x)=x(x-1). For value x∈(-[tex]\infty[/tex],m], f(x)≥[tex]\frac{-8}{9}[/tex]. What is the value of m?​

the smallest value m=[tex]\frac{7}{3}[/tex].

Given that, for x∈(0,1], f(x)=x(x-1)

Let's plot this function

We'll go from 0 to 1 and let's graph what this function is look like

f(x)=x(x-1) will be a parabola and it's going to go between the points (0,0) and (1,0).

Will graph it in figure1 we can see.

The minimum value of this parabola is right in the middle and it is

f(0.5)=-0.25

Now, let try and graph the function in the other intervals

We know f(x+1)=2f(x)

Now we can evaluate couple of points

f(0+1)=2f(x)

f(0.5+1)=2f(0.5)

f(1+1)=2f(1)

And we quickly deduce that the graph in(1,2] is twice the height of the graph in (0,1].

We will graph a parabola in figure 2 we can see.

This parabola is going to have a graph of 2 between its twice as tall and this can be multiplied by the quantity of (x-1) multiplied by the quantity of (x-2).

It is 2(x-1)(x-2).

We graph next interval twice the height of the graph in the interval from 2 to 3. We can see the graph in figure3.

This parabola will have an equation 4 times the quantity of (x-2) and (x-3).

We can also shift this graph to the left and it going to be half the height.

Looking at this graph we want f(x)≥[tex]\frac{-8}{9}[/tex].

In the 3 interval from 2 to 3 its going to intersect .

we want to look for the points where f(x)≥[tex]\frac{-8}{9}[/tex].

We are looking for m which f(x)≥[tex]\frac{-8}{9}[/tex].

parabolas equation =[tex]\frac{-8}{9}[/tex].

4(x-2)(x-3)=[tex]\frac{-8}{9}[/tex]

[tex]x^{2} -5x+6=\frac{-2}{9} \\(x-\frac{7}{3})(x-\frac{8}{3})=0\\ x= \frac{7}{3},\frac{8}{3}\\\\[/tex]

Therefore, We will take the smallest value m=[tex]\frac{7}{3}[/tex].

To learn more about function visit:https://brainly.com/question/12183687

#SPJ1

Dot arranges flowers into vases of roses and daisies. If there are 100 roses and 60 daises. what is the greatest number number of flowers in each group?

Answers

The greatest number of flowers in each group is 20.

What is the greatest number number of flowers in each group?

In order to determine the greatest number number of flowers in each group, the highest common factor of 100 roses and 60 daises have to be determined.

The factor of a number is a number that divides a number perfectly into whole numbers. The highest common factor is the highest factor that is common to two or more numbers

Factors of 100 = 1, 2, 4, 5, 10, 20, 25, 50 and 100.

Factors of 60 = 1, 2, 3, 4, 5, 6, 10, 12, 15, 20, 30 and 60.

Common factors of 100 and 60 - 1, 2, 5, 10, 20

Highest common factor = 20

To learn more about highest common factor, please check: https://brainly.com/question/26073850

#SPJ1

The median annual salary for an event planner is $60,900. If the event planner pays an
8% income tax, how much money in dollars and cents will the event planner pay for
income tax?
Sunt

Answers

Answer:

$4,872.00

Step-by-step explanation:

Convert 8% into a decimal and multiply

$60,900.00 x 0.08 = $4,872.00

Three 6" x 6" and two 8" x 8" electrical boxes are installed in a wall between two studs. There are _ square inches of wall area taken up by the boxes.

Answers

Answer: 236 square inches

Step-by-step explanation:

Three 6" x 6" and two 8" x 8" electrical boxes are installed in a wall between two studs

The answer is 236

Divide. (6x³ + 2x² + 3x −4) ÷ (x² + 3x +4)​

Answers

6x³ + 2x² + 3x - 4 is not exactly divisible by x² + 3x - 4.

Given,

The expression : 6x³ + 2x² + 3x - 4

We have to divide the given expression by x² + 3x - 4.

Let's take 6x³ + 2x² + 3x - 4 as p(x) and x² + 3x - 4 as g(x).

If p(x) is divisible by the factors of g(x), then p(x) is divisible by g(x).

So, let's factorize  x² + 3x - 4

x² + 3x - 4 = (x - 1) (x + 4)

Now,

p(x) = 6x³ + 2x² + 3x - 4

By using remainder theorem,

If p(1) = 0 and p(4) = 0, then p(x) is divisible by g(x).

So,

p(1) = 6 × 1³ + 2 x 1² + 3 x 1 -4

p(1) = 6 + 2 + 3 - 4

p(1) = 11 - 4

p(1) = 7

Next,

p(4) = 6 x 4³ + 2 x 4² + 3 x 4 - 4

p(4) = 6 x 64 + 2 x 16 + 12 - 4

p(4) = 384 + 32 + 12 - 4

p(4) = 424

Here, p(1) and p(4) is not 0.

So, p(x) is not divisible by g(x).

That is, 6x³ + 2x² + 3x - 4 is not exactly divisible by x² + 3x - 4.

Learn more about divisibility here:

https://brainly.com/question/1591815

#SPJ1

The subtraction problem 4/5 - 2/7 can be rewritten as the addition problem _[blank].
Enter your answer as the expression that correctly fills in the blank.
Enter fractions formatted like this: 3/14

Answers

It can be written as 13/35.

how do we rewrite a fraction.

Divide the numerator by the denominator

Write down the whole number result

Use the remainder as the new numerator over the denominator. This is the fraction part of the mixed number.

A number that will divide evenly into both the numerator and denominator so it can be reduced, or

The numerator must be greater than the denominator, (an improper fraction), so it can be converted to a mixed number

4/5-3/7

⇒4×7-3×5/ 35     ( 5 and 7 have no common divisor).

⇒ 28- 15/35

⇒13/35

To know more about fractions.

visit-:https://brainly.com/question/10354322

#SPJ9


What is the amount of the current ratio?

Answers

Answer:4.98

Step-by-step explanation:

trust me

Angela is arranging 32 framed pictures on a wall. How
can she arrange her pictures in equal rows? Find all
possible arrangements.

Answers

Answer:

1×32

2×16

4×8

8×4

16×2

32×1

Step-by-step explanation:

This question about arranging pictures is just trying to get you to explore the factors of 32, maybe for practicing the math ideas of factoring, or area, or max/min, or matrices.

Think of what numbers go into 32:

1, 2, 4, 8, 16, 32

Pair them up!

1×32 and 32×1 will look differently if they are pictures on a wall. So we'll list them separately. Then there's also 2×16 and 16×2, and 4×8 and 8×4.

write an equation of a line passing through the point (-1,3) and parallel to AB with A(3,-5) and B (-2,15)

Answers

Answer:

y = -4x - 1

Step-by-step explanation:

Let's first find the equation of the line segment AB. The equation of a line in slope-intercept form is y = mx + b where m represents the slope and b represents the y-intercept. Start by finding the slope.

Finding the Slope of AB

The equation for slope is:  [tex]\frac{y_2-y_1}{x_2-x_1}[/tex]. These variables represent any pair of coordinates on the line. In this case since the two points chosen are A and B, thus:

[tex]x_2=-2\\x_1=3\\y_2=15\\y_1=-5[/tex]

If we plug these values into the equation, we get:

[tex]\frac{15-(-5)}{-2-3}=\frac{20}{-5}=-4[/tex]

Now we need to find the equation of a line that passes through the point (-1, 3) and is parallel to AB. If two lines are parallel they share the same slope. The equation of the line parallel to AB is y = -4x + b. We can plug in the coordinate (-1, 3) into the equation to solve for b.

Solving for the y-intercept

[tex]y = mx+b\\3 = -4(-1) +b\\3=4+b\\\text{Subtract 4 from both sides}\\b=-1[/tex]

Therefore the equation of the line that passes through the point (-1, 3) and is parallel to AB is y = -4x - 1.

Find the angle of elevation of the sun from the ground to the top of a tree when a tree that is 10 yards tall casts a shadow 14 yards long. Round to the nearest degree.

Answers

The angle of elevation of the sun from the ground to the top of a tree is 36°.

We are given that:

The sun from the ground to the top of a tree when a tree that is 10 yards tall casts a shadow 14 yards long.

Now by using the trigonometric identity .

tan θ = P / B

As figure is  given below .

AB =  Perpendicular = 10 yards

BC =  Base = 14 yards

Putting all the values in the trigonometric identity .

tan θ = 10 / 14

θ = 36°

Therefore, the angle of elevation of the sun from the ground to the top of a tree is 36°.

Learn more about angle of elevation here:

https://brainly.com/question/1851601

#SPJ9

Target had 8x10⁹ dollars in sales last year. Explain how to find the product 8x10⁹.

Answers

Using scientific notation, The value of  8x10⁹ is 8000000000

How should a scientific notation be written?

In scientific notation, the correct format is an x 10 b, where an is an integer or decimal number whose absolute value is more than or equal to one and less than ten, or 1 |a| 10. In order for the scientific notation to be mathematically identical to the original number, b must be a power of 10.

The given problem is written in scientific notation,

The multiplication of the given number will be-

8x10⁹ = 8000000000

To learn more about scientific notation from given link

https://brainly.com/question/1767229

#SPJ9

Use the graph of f to determine each of the following.
(a) the domain of f
(b) the range of f
(c) the zero(s) of f
(d) f(-3.5)
(e) the intervals on which f is increasing
(f) the intervals on which f is decreasing
(g) the values for which f(x) ≤0
(h) any relative maxima or minima
(i) the value(s) of x for which f(x) = 3
() Is f(0) positive or negative?

Answers

(a) the domain of f would be an interval (−3,6]

(b) the range of f would be an interval (−6,3]

(c) the zero(s) of f are x = −0.5 and x=6.

(d) the value of f(-3.5) would be 2

(e) the intervals at (−3,3) which f is increasing

(f) the intervals at (3,6] which f is decreasing

(g) the values are (−3,−0.5] for which f(x) ≤0

(h) x = 3 which is a relative maximum point

(i) the value of x is 3 for which f(x) = 3

(j) the function would be positive if f(0)

What are the domain and range of the function?

The domain of the function includes all possible x values of a function, and the range includes all possible y values of the function.

The domain of the function shown in the graph is given by the set of values x that has a point on the graph.

In other words, by launching the graph onto the x-axis we will cover the interval (−3,6]

The range of the function shown in the picture is given by the set of values y that has a point on the graph.

In other words, by launching the graph onto the y-axis we will cover the interval (−6,3]

The zeros of the function are the x-intercepts of the function, so the zeros are x = −0.5 and x=6.

The value of f(-3.5) is given by the y- coordinate of the point of intersection of the graph with the vertical line x=-3.5 So, f(-3.5)≈2.

The function is increasing whenever the y- coordinate increases if we trace the graph in the direction of positive x values.

So, the graph is increasing on (−3,3), where x=3 corresponds to the x- coordinate of the turning point on the graph.

The function is decreasing whenever the y- coordinate decreases if we trace the graph in the direction of positive x values.

So, the graph is decreasing on (3,6].

The values of x where f(x) ≤0, correspond to the points where the graph is below the x-axis, so the values of x for which f(x) ≤0 are (−3,−0.5].

The relative maxima or minima are the x-coordinate points where the continuous function has a turning point, like x=3 which is a relative maximum point

.

There are no relative minimum points for the given function.

The value of x for f(x)=3 is the x- coordinate of the point of intersection of the graph with the horizontal line y=3

So, x≈3.

we will determine the  y = coordinate of the point of intersection of the graph with the vertical line

x=0, which is above 2, so f(0) is positive.

Therefore, the domain of the function is interval (−3,6], and the range of the function is interval (−6,3].

Learn more about the domain and the range here:

brainly.com/question/21027387

#SPJ1

the ratio of red cars to blue cars in a parking lot was 5:3. If there was 40 red cars how many blue cars were there?

Answers

The ratio of red cars to blue cars in a parking lot was 5:3, total blue cars are 24.

What is ratio?

A ratio in mathematics shows how many times one number is included in another. For instance, if a dish of fruit contains eight oranges and six lemons, the ratio of oranges to lemons is eight to six. The ratio of oranges to the overall amount of fruit is 8:14, while the ratio of lemons to oranges is 6:8.

Given, the ratio is 5:3.

Total red cars = 40

⇒ 5x = 40

⇒ x  = 8

Blue cars  = 8 × 3

= 24

To learn more about ratio from given link

https://brainly.com/question/12024093

#SPJ9

A newspaper advertisement offers a $9,000 car for nothing down and 36 easy monthly payments of $317.50. What is the simple interest rate?

_____%

Answers

A=$11,430 ; I=$2,430

Let s(t)=[tex]\sqrt{x+16}[/tex] be the position function. Use the Definition to find the velocity at t=0.

Answers

The velocity at t = 0 will be 4 meters per second.

What is a function?

A function is defined as the expression that set up the relationship between the dependent variable and independent variable.

A function in mathematics from a set X to a set Y assigns exactly one element of Y to each element of X. The sets X and Y are collectively referred to as the function's domain and codomain, respectively.

Given function is s(t) = √ ( t + 16 ).

The value of the function is calculated as below:-

s ( t ) = √ ( t + 16 ).

s( 0 ) = √ ( 0 + 16 ).

s( 0 ) = √ ( 16 ).

s( 0 ) = 4

Therefore, the velocity at t = 0 will be 4 meters per second.

To know more about functions follow

https://brainly.com/question/25638609

#SPJ1

Hay 12 estudiantes en una clase que se presentan en el concierto de la escuela. Esto representa el 40% de la clase. ¿Cuántos estudiantes hay en la clase?

Answers

Mediante el uso de porcentajes, concluimos que la clase tiene una población total de 30 estudiantes.

¿Cuál es la cantidad de estudiantes de la clase?

En este problema tenemos a 12 estudiantes procedentes de una clase que participan en un evento cultural de su escuela y también se conoce que equivalen al 40 por ciento de su clase. El total de estudiantes de la clase se obtiene al dividir el número de estudiantes por el porcentaje:

x = 12 / 40 %

x = (12 · 100) / 40

x = 30

La clase tiene una población total de 30 estudiantes.

Para aprender más sobre porcentajes: https://brainly.com/question/23935688

#SPJ1

A gardener makes a new circular flower bed. The bed is twelve feet in diameter. Calculate the circumference and the area of the
circular flower bed.
A. circumference = 12 feet, area = 12 square feet
B. circumference = 6 feet, area = 36:
square feet
C. circumference = 12
feet, area = 144 square feet
D. circumference = 12 feet, area = 36 square feet

Answers

Answer:

Step-by-step explanation: I think the answer would be B or D they seem the most logical

Cant solve this problem

Answers

a) The graph will have a "V" shape, which opens upwards.

b) The horizontal shift of 1 unit to the left.

c) The vertical shift is of 5 units downwards.

How to describe the graph of the absolute value function?

Here we have the absolute value function:

f(x) = |x + 1| - 5

First we want to describe the graph shape, we know that it is an absolute value function, so it will have a "V" shape.

The arms will make an angle of 45° with the vertical and horizontal (because the slope is 1)

And it will open upwards, because the slope is positive.

Now, remember that if the parent function is:

f(x).

A translation is written as:

g(x) = f(x + a) + b

Where a is the horizontal translation and b is the vertical translation, such that:

a < 0, translation to the right.a > 0, translation to the left.b > 0 translation up.b < 0 translation down.

In this case we have:

f(x) = |x + 1| - 5

So this is a translation of 1 unit to the left and 5 units down of the parent absolute value function.

b) The horizontal shift of 1 unit to the left.

c) The vertical shift is of 5 units downwards.

If you want to learn more about absolute value functions:

https://brainly.com/question/3381225

#SPJ1

Help please with my math

Answers

The percentage of the actual salary that actual mortgage is is 30%.

What is the percentage of actual mortgage?

Percentage is the fraction of an amount that is expressed as a number out of hundred. Percentage is a measure of frequency. The sign used to show percentage is %.

In order to transform a number into a percentage, express it as a fraction of a total amount and multiply by 100. So, actual mortgage would be divided by actual budget, the fraction would then be multiplied by 100.

Percentage of actual salary that is actual mortgage  = (actual mortgage / actual salary) x 100

(1080 / 3600) x 100 = 30%

To learn more about percentages, please check: https://brainly.com/question/25764815

#SPJ1

The mass of flour in a bag is about 141.6 grams. You add about 10.19 grams of flour to the bag. What is the mass of flour in the bag to the nearest tenth of a gram

Answers

The tenth of a gram closest to it is 151.79 grams.

The flour's mass is 141.6 grams.

10.19 grams are added.

The amount of flour in the bag =141.6+ 10.19

= 151.79.

A physical quantity is mass. The weight of a body serves as the unit of mass measurement. It can be measured in normal mass quantities such grams, kilograms, and pounds. Although grams are used to measure smaller objects, the kilogram is the SI unit of mass.

In the metric measurement system, a gram is a unit of weight that is one thousandth of a kilogram. When something is extremely light, its weight is measured in grams.

Therefore, 151.79 grams of flour, to the nearest tenth of a gram, are in the bag.

To learn more about mass, visit:

brainly.com/question/941818

#SPJ1

Find the range of f(x)=4x+2 for the domain {–2, –1, 1, 4}. Your answer:

Answers

The range of the function, f(x)=4x+2 for the domain {–2, –1, 1, 4} includes (-6, -2, 6, 18).

What is the range of a function?

The range of the function,  f(x)=4x+2, includes all the possible output values of the function.

What is the domain of a function?

The domain of a function,  f(x)=4x+2, is the set of all possible input values (domain {–2, –1, 1, 4}) of the function.

Remember that a function is a mathematical expression that equates an independent variable with a dependent variable.

Data and Calculations:

f(x)=4x+2

Domain =  {–2, –1, 1, 4}

The range for -2:

f(x)=4x+2 = 4(–2) + 2 = -6

Range for -1:

f(x)=4x+2 = 4(–1) + 2 = -2

Range for 1:

f(x)=4x+2 = 4(1) + 2 = 6

The range for 4:

f(x)=4x+2 = 4(4) + 2 = 18

Thus, the range of the function, f(x)=4x+2 for the domain {–2, –1, 1, 4} includes (-6, -2, 6, 18).

Learn more about the range, domain, and function at https://brainly.com/question/1942755

#SPJ1

sergio was shopping for online stocks in nvidia. he purchased 2,000 shares at 4 dollars each. if sergio plans for the stockts to triple in value by the end of next year what will his shares be worth in total?​

Answers

The total worth of shares that he has is 24000$

To calculate earnings per share, take a company's net income and deduct preferred dividends from it. Divide the total by the average number of outstanding common shares.

Given Sergio purchased 2000 shares at four dollars each

the cost price of shares is 4$ each

Therefore total shares worth he has  = 2000 * 4

                                                             = 8000 $

He now expects the stock to triple in value by the end of next year.

total amount of shares he owns = 8000 * 3

                                                      = 24000$

Therefore, the total amount of shares he owns is 24000$

Learn more about the calculation of shares here:

https://brainly.com/question/1179405

#SPJ9

Evaluate. Write your answer as a fraction in simplest form. 1/2 + 7/18 + 5/6 please help me asap​

Answers

Let’s begin by converting into the common denominator of 18:

1/2 = 9/18 since 2 • 9 = 18 and 1 • 9 = 9

5/6 = 15/18 since 5 • 3 = 15 and 6 • 3 = 18

Now we can add accordingly:

9/18 + 7/18 + 15/18

9 + 7 = 16

16 + 15 = 31

Our final answer is 31/18, but we aren’t done. We still have to convert to simplest form:

18 • 1 = 18

18 + 13 = 31

In terms of simplest form conversion, our mixed number conversion is 1 13/18.

Our answer is 1 13/18.

Answer:

31/18 which is changed to 1³¹/¹⁸

Step-by-step explanation:

9/18+7/18+15/18 I found the lcm or the lowest common multiple of all the three and multiplied it by each fraction first then u can add all of them and put it over 18 or add each

Union and Intersection of sets
[tex] \\ \\ [/tex]
A. Indicate whether the statement is true or false.

1.) Union of two sets is the set that consists all the elements belonging either A or to B, or to both.

2.) If A = {2, 3, 4} and B = {3, 4, 5, 6} then A ∪ B = {1, 2, 3, 4, 5, 6}

3.) If M = {a, e, i, o, u} and N = {l, o, v, e} then M ∩ N = {o, e}​

Answers

Answer:

Step-by-step explanation:

1.true

2.true

3.true

Answer:

1)  True

2)  False

3)  True

Step-by-step explanation:

Set Notation

[tex]\begin{array}{|c|c|l|} \cline{1-3} \sf Symbol & \sf N\:\!ame & \sf Meaning \\\cline{1-3} \{ \: \} & \sf Set & \sf A\:collection\:of\:elements\\\cline{1-3} \cup & \sf Union & \sf A \cup B=elements\:in\:A\:or\:B\:(or\:both)}\\\cline{1-3} \cap & \sf Intersection & \sf A \cap B=elements\: in \:both\: A \:and \:B} \\\cline{1-3} \sf ' \:or\: ^c & \sf Complement & \sf A'=elements\: not\: in\: A \\\cline{1-3} \sf - & \sf Difference & \sf A-B=elements \:in \:A \:but\: not\: in \:B}\\\cline{1-3} \end{array}[/tex]

Question 1

The union of two sets is denoted by the symbol ∪.

The union of two sets is the set that comprises all elements in A or B or both.

Question 2

Given sets:

A = {2, 3, 4}B = {3, 4, 5, 6}

[tex]\begin{aligned} \implies \sf A \cup B & =\sf \{ 2, 3, 4\} \cup \{3, 4, 5, 6 \}\\& =\sf \{ 2, 3, 4, 5, 6\}\end{aligned}[/tex]

Question 3

Given sets:

M = {a, e, i, o, u}N = {l, o, v, e}

[tex]\begin{aligned}\implies \sf M \cap N & =\sf \{ a, e, i, o, u\} \cap \{l, o, v, e \}\\& =\sf \{ o, e\}\end{aligned}[/tex]

Learn more about set notation here:

https://brainly.com/question/28356437

https://brainly.com/question/28353607

-4ca: use a = 2. and c = -5

Answers

Answer:

40

Step-by-step explanation:

For this equation, all we do is simply plug in the values.

-4 (c) (a)

-4 (-5) (2)

-4*-5*2

Now, multiply, and you get your answer.

-4 * -5 = 20

20 * 2 = 40

40 is your answer

Today, Sarah counted all of the wheels of all of the cycles in the shop. She found that there were 30 wheels in all. There were the same number of bikes as there were trikes. How many bikes were there? How many trikes were there? Show how you figured it out.​

Answers

Answer:

Step-by-step explanation:

Bicycles have two wheels whereas Trikes  have 3 wheels. Let x be the number of bicycles2x=30[tex]\frac{2x}{2} =\frac{30}{2} \\x=15[/tex]There were 15 bicyclesLet y be the number of trikes3x=30 since there were same number of trikes as bicycles[tex]\frac{3y}{3} =\frac{30}{3} \\y=10[/tex]There were 10 Trikes

6.62 inches is more than 6.4 inches ?​

Answers

Answer is yes 6.62 inches is more than 6.4 inches

Step by step

6.62 is 6 62/100 in a fraction form
6.40 is 6 40/100 in a fraction form

62 is larger than 40 in both decimal and in fraction form

(35 POINTS!!) Given f of x is equal to 1 over the quantity x minus 3 and g of x is equal to the square root of the quantity x plus 3, what is the domain of f (g(x))?

[–3, ∞)
[–3, 6) ∪ (6, ∞)
(–∞, 3) ∪ (3, ∞)

Answers

Answer:

the second option

[–3, 6) ∪ (6, ∞)

Step-by-step explanation:

f(x) = 1/(x - 3)

the domain is every value of x that does not turn the denominator to 0. so, x <> 3.

g(x) = sqrt(x + 3)

the domain is every value of x that does not turn (x+3) into a negative value. so, x >= -3.

f(g(x)) = 1/(sqrt(x + 3) - 3)

the domain is all values of x that are valid for g(x) AND that do not generate range values of g(x) that violate the domain of f(x).

so, it is (x >= -3) AND (sqrt(x + 3) <> 3).

and sqrt(x + 3) = 3, if (x + 3) = 9.

that means x = 6.

so, 6 is forbidden, every other value of x is allowed here.

therefore, the domain of f(g(x)) is

(x >= -3) AND (x <> 6)

and that is equivalent to

[-3, 6) u (6, infinity)

everything larger or equal to -3 and different from 6.

Other Questions
Convert 3/7 to percent what is the story totally free about Find the measure of each angle specified below. Pablo folds a straw into a triangle with side lengths of 4x23 inches, 4x22 inches, and 4x21 inches. Which expression can be used to find the perimeter of the triangle, and what is the perimeter when x = 1.5?12x2 6; 21 inches12x2; 27 inches12x2 6; 30 inches12x2; 36 inches A28% income tax on a 80,000 income is disinfectant spray safe to use everyday Determine whether each sequence is arithmetic. If so, identify the common difference and find the 32 nd term of the sequence. 2,5,9,14,. . . . . How did physical geography affect the development of theeconomies of Mayan city-states?Being located in central Mexico gave the Maya accessto rich deposits of silver ore.Occupying a high mountain valley allowed for theintroduction of potato agriculture.Close proximity to Teotihuacn allowed for theformation of strong trade connections.Access to rubber trees ensured control of whoparticipated in the ball game (please quick))Click to review the online content. Then answer the question(s) below, using complete sentences. Scroll down to view additional questions.Online Content: Site 1Explain the reason countries trade with each other. (Site 1) Find two - 3 - digits which are divisible by 8 and 11? Kary took 4 courses last semester: History, Biology, Calculus, and Physics. The means and standard deviations for the final exams, and Kary's scores are given in the table below. Convert Kary's score into z scores.Subject Mean Stand. dev. Kary's score Kary's z scoreHistory 53 16 69 Biology 77 10 74.5 Calculus 70 12 88 Physics 60 14 67 which step of the critical process requires students evaluate symbols found in the work of art?A.ContextB. FormC. IconographyD. Medium To mail a letter to Vienna, Austria the post office charges a flat rate of $8.00 and an additional $0.75 for every ounce the letter weighs. The cost of mailing a letter isdetermined by the equation y = 0.75x+8.00, where y is the cost of the postage and x is the weight of the letter in ounces.Step 2 of 2: Use the graph to estimate to the nearest quarter of a dollar ($0.25) the cost to send a letter that weighs 5 ounces.Enable Zoom/PanPrice for Postage1510S510Weight (oz)15 A car is traveling at a constant speed of 65 miles per hour. Drag each list or statement below to show whether it describes the independent quantity or the dependent quantity.heres the answer Below zero winter temperatures, rivers and lakes are frozen Here are some values of sequence . Write a recursive definition for the sequence. 1,4 3,7 7,13 Drag the tiles to the boxes to form correct pairs.Match the pairs of equivalent expressions. Find x and the measure of each side.R S T is equilateral. R S is three more than four times x, S T is seven more than two times x , and T R is one more than five times x . The first mall in mumbai, india was successful. Builders proceeded to construct three more right away. However, they have more than 50% vacancies because they lack customers. The builders failed to properly account for which challenge of operating in developing countries?. A car initially has a velocity of 29 m/s starts to increase its velocity at a constant rate of 2.4 m/s2, for a distance of 22 m. What is its final velocity?